Prove a limit using the mean value theorem

Click For Summary
To prove that lim_x→infty(√(x+5)-√(x))=0 using the mean value theorem, the function f(x)=√x is considered. By applying the theorem, it is established that there exists a point c between x and x+5 where the derivative f'(c)=1/(2√c) approaches 0 as c approaches infinity. The expression (f(b)-f(a))/(b-a) simplifies to f'(c), leading to the conclusion that the limit of the difference also approaches 0. Thus, the reasoning confirms that the limit is indeed 0, as both the numerator and the derivative vanish in the limit. This demonstrates the application of the mean value theorem effectively in proving the limit.
Charlotte87
Messages
18
Reaction score
0
I am supposed to use the mean-value theorem to show that lim_x→infty(√(x+5)-√(x))=0.

Can anyone help me solving this problem?

I have tried to set up the mean value theorem, but i just do not know how to proceed.
 
Physics news on Phys.org
It is rather obvious even without the mean value theorem. Just multiply and divide by the conjugate.
 
I also believe that the limit is obvious, but the exercise here is to prove it using the mean value theorem.
 
The "mean value theorem" says that, for f continuous on [a, b] and differentiable on (a, b), there exist c between a and b such that
\frac{f(b)- f(a)}{b- a}= f'(c).

Take f(x)= x1/2, b= x+5, a= x. Of course, as x goes to infinity, so do both x and x+5 so a number "between them", c, must also go to infinity. What can you say about f'(c) as c goes to infinity?
 
So, if I write that ((f(b)-f(a))/b-a)=f'(c), and takes the value as you say. I know that f'(x)=1/2sqrt(x). The limit of this when x goes to infinity, is 1 divided by an infinity large number which is 0.

Am I right with my reasoning below: this proves my limit, since lim(f(b)-f(a))=lim(f'(c)*(b-a))=0 since f'(c)=0?
 
Question: A clock's minute hand has length 4 and its hour hand has length 3. What is the distance between the tips at the moment when it is increasing most rapidly?(Putnam Exam Question) Answer: Making assumption that both the hands moves at constant angular velocities, the answer is ## \sqrt{7} .## But don't you think this assumption is somewhat doubtful and wrong?

Similar threads

  • · Replies 1 ·
Replies
1
Views
957
  • · Replies 5 ·
Replies
5
Views
1K
  • · Replies 11 ·
Replies
11
Views
2K
Replies
3
Views
2K
  • · Replies 6 ·
Replies
6
Views
1K
  • · Replies 23 ·
Replies
23
Views
2K
  • · Replies 1 ·
Replies
1
Views
2K
  • · Replies 12 ·
Replies
12
Views
1K
Replies
4
Views
2K
  • · Replies 12 ·
Replies
12
Views
3K